A vs. D
Hi LSAT Max! When I completed this passage for the diagnostic exam, I went with answer choice ...
Julie-V on August 14, 2019
  • December 2004 LSAT
  • SEC3
  • Q5
4
Replies
Question 9
Hello, would someone be able to explain this question to me?
Samir-Ghani on July 27, 2019
  • December 2004 LSAT
  • SEC3
  • Q9
1
Reply
Question 12
Can someone explain this question to me?
Samir-Ghani on July 27, 2019
  • December 2004 LSAT
  • SEC3
  • Q12
1
Reply
Explanation for D
Hello! I chose D because of lines 8-11 because in my head it sounded like it answered the que...
Julie-V on July 13, 2019
  • December 2004 LSAT
  • SEC3
  • Q22
1
Reply
Explanation
I was stuck between B and C. Why is B correct?
Marissa-Avnaim on June 20, 2019
  • December 2004 LSAT
  • SEC3
  • Q24
1
Reply
Correct Answer
Why is A incorrect?
Lizzie-Annerino on June 3, 2019
  • December 2004 LSAT
  • SEC3
  • Q7
1
Reply
Why is it not B?
Based on the tone, I thought the author was very dismissive of the critics, not in reasoned disag...
Jessica-Killeen on March 20, 2019
  • December 2004 LSAT
  • SEC3
  • Q13
2
Replies
Extension Sentences
Hi, could you please explain why E is the correct answer here. And more generally, what is the be...
Richmond on February 21, 2019
  • December 2004 LSAT
  • SEC3
  • Q11
3
Replies
Answer
What's the answer and why. Thanks!
Ceci on February 9, 2019
  • December 2004 LSAT
  • SEC3
  • Q2
2
Replies
Why is A incorrect?
Why is A incorrect?
Shiyi-Zhang on February 8, 2019
  • December 2004 LSAT
  • SEC3
  • Q16
1
Reply
Answer
What's the answer. Thanks!
Ceci on February 6, 2019
  • December 2004 LSAT
  • SEC3
  • Q27
2
Replies
Why is answer C incorrect?
Could you please explain why C is incorrect?
ecal23 on November 14, 2018
  • December 2004 LSAT
  • SEC3
  • Q26
1
Reply
please explain
I chose answer choice C. could you please explain why this answer choice is incorrect
kdehoyos on July 31, 2018
  • December 2004 LSAT
  • SEC3
  • Q12
1
Reply
Answer Choice C
Why is C incorrect?
EmilyMarieMenendez on July 24, 2018
  • December 2004 LSAT
  • SEC3
  • Q1
1
Reply
Answer choice C?
I can't figure out as to why C is not the correct answer. It seems as though that is the authors ...
AnkitM on July 12, 2018
  • December 2004 LSAT
  • SEC3
  • Q10
3
Replies
Why not E?
From the passage it seemed that the author would be in favor of the local community taking charge...
joshuavt on May 28, 2018
  • December 2004 LSAT
  • SEC3
  • Q5
2
Replies
Why C?
Could you please explain the logic behind choosing C instead of D?
joshuavt on May 27, 2018
  • December 2004 LSAT
  • SEC3
  • Q4
1
Reply
Question
Why is A wrong and C right?
meisen on May 20, 2018
  • December 2004 LSAT
  • SEC3
  • Q25
2
Replies
Hellp
Hello, I cannot understand why answer choice A was incorrect. Please help me to understand, thanks.
Kane44 on May 9, 2018
  • December 2004 LSAT
  • SEC3
  • Q1
1
Reply
Why isn't it D
So, I'm confused as to why this isn't D. The lines specifically call out the vague language as a ...
knoxygirl on May 16, 2017
  • December 2004 LSAT
  • SEC3
  • Q21
1
Reply